一些极限的证明

极限的计算是建立在已知的若干极限和极限运算法则基础上的。除此之外,还有很多不能直接计算出来,需要证明的极限。这里列出一些。

  1. $\lim_{n\rightarrow\infty} \sqrt[n]{n}=1$

    证明:令$x_n=\sqrt[n]{n}$. 因为 $n$ 是正整数且趋于无穷,故考虑 $n>1$,所以 $x_n>1$. 则可设 $x_n=1+t_n$,那么 $t_n=\sqrt[n]{n}-1>0$. 根据二项式定理, $$ n=\left(1+t_{n}\right)^{n}=1+n t_{n}+\frac{n(n-1)}{2} t_{n}^{2} + 一些正项, $$ 所以 $$ \frac{n(n-1)}{2} t_{n}^{2}<n \Longrightarrow t_{n}^{2}<\frac{2}{n-1} \Longrightarrow t_{n}<\frac{\sqrt{2}}{\sqrt{n-1}}. $$ 则可得到 $$ 1<x_{n}=1+t_{n}<1+\frac{\sqrt{2}}{\sqrt{n-1}} \Longrightarrow 1<x_{n}<1+\frac{\sqrt{2}}{\sqrt{n-1}}. $$ 计算可得 $$ \lim_{n\rightarrow\infty} 1= 1 = \lim_{n\rightarrow\infty} \left(1+\frac{\sqrt{2}}{\sqrt{n-1}}\right)=1. $$ 故运用夹逼定理(Squeeze Theorem)可得 $\lim_{n\rightarrow\infty} \sqrt[n]{n}=1$.

  2. $\lim\limits_{n \rightarrow \infty} \sqrt{1+2 \sqrt{1+3 \sqrt{1+\cdots \sqrt{1+(n-1) \sqrt{1+n}}}}}=3$

    这是一个嵌套根式。拉马努金曾得到这样的公式: $$ x + n + a =\sqrt{ ax +( n + a )^{2}+ x \sqrt{ a( x + n )+( n + a )^{2}+( x + n ) \sqrt{\cdots}}}.$$ 取 $ x=2,n=1,a=0$,即可得该极限为3。不过这里要给出的是一个完整的证明。

    令 $$a_{m, n}=\sqrt{1+m \sqrt{1+(m+1) \sqrt{1+\cdots+(n-1) \sqrt{1+n}}}},\quad (1\leqslant m \leqslant n),$$ 其中 $n$ 看作已知,$m$ 除了出现在表达式里以外,这样理解方便一些:从里到外共 $ n-m+1$ 个根号,比如 $a_{n,n}=\sqrt{1+n}$。那么就有递推关系: $$ a_{k, n}=\sqrt{1+k a_{k+1, n}}. $$ 而待求的极限则可以写作: $$ \lim\limits_{n\to\infty} a_{2,n}.$$ 下面会计算 $ \lim\limits_{n\to\infty} a_{m,n}$,然后取 $m=2$ 到得结果。 使用的是夹逼准则,首先证明上界满足: $$ a_{m,n}<m+1. $$ 前面已经说过 $a_{n,n}=\sqrt{1+n}$,因为 $n$ 是正整数且趋于无穷,显然有 $a_{n,n}<n+1$。那么用归纳法,设 $a_{k+1,n}<k+2 $,只需证明此时有 $a_{k,n}<k+1 $。证明如下: $$a_{k, n}=\sqrt{1+k a_{k+1, n}}<\sqrt{1+k(k+2)}=\sqrt{k^{2}+2 k+1}=k+1$$ 此时已经求出 $a_{m,n}$ 的一个上界 $m+1$,于是有 \begin{equation} 0<m+1-a_{m,n}. \label{eq:upper} \end{equation} 讨论下界需要用到另外一个结论。在上面的分析中,构造的通项如果是 $$b_{m,n}=\sqrt{1+m \sqrt{1+(m+1) \sqrt{1+\cdots+(n-1) \sqrt{1+n^{2}+2 n}}}},$$ 则可以验证 $b_{n,n}=\sqrt{1+n^2+2n}=n+1$,再用归纳法可得 $$m+1=\sqrt{1+m \sqrt{1+(m+1) \sqrt{1+\cdots+(n-1) \sqrt{1+n^{2}+2 n}}}}.$$ 结合这个式子,用平方差公式,有 $$ \begin{aligned} m+1 - a_{m,n} & = \sqrt{1+m \sqrt{1+(m+1) \sqrt{1+\cdots+(n-1) \sqrt{1+n^{2}+2 n}}}} - \sqrt{1+m \sqrt{1+(m+1) \sqrt{1+\cdots+(n-1) \sqrt{1+n}}}} \\ & = \frac{m \left( \sqrt{1+(m+1) \sqrt{1+\cdots+(n-1) \sqrt{1+n^{2}+2 n}}} + \sqrt{1+(m+1) \sqrt{1+\cdots+(n-1) \sqrt{1+n}}} \right)}{\sqrt{1+m \sqrt{1+(m+1) \sqrt{1+\cdots+(n-1) \sqrt{1+n^{2}+2 n}}}} + \sqrt{1+m \sqrt{1+(m+1) \sqrt{1+\cdots+(n-1) \sqrt{1+n}}}}} \\ & = \frac{m(m+2-a_{m+1,n})}{m+2+a_{m,n}} \\ & < \frac{m}{m+2}(m+2-a_{m+1,n}) \end{aligned} $$ 重复运用这个不等式,得到 $$ \begin{aligned} m+1 - a_{m,n} &< \frac{m}{m+2}(m+2-a_{m+1,n})<\cdots<\frac{m(m+1)\cdots(n-1)}{(m+2)(m+3)\cdots(n+1)}(n+1-a_{n,n}) \\ & < \frac{m(m+1)(n+1-\sqrt{n+1})}{n(n+1)} \\ & < \frac{m(m+1)}{n} \end{aligned} $$ 结合前面已经得到的 \eqref{eq:upper},有 $$ 0<m+1-a_{m,n}<\frac{m(m+1)}{n}. $$ 当 $ n \to \infty $时,两边都趋于0,所以 $$ \lim\limits_{n\to\infty} a_{m,n} = m+1. $$ 取 $m=2$,即可得要证明的极限.


本文作者: Jing Han
本文链接: http://www.jhanmath.com/?p=41
版权声明: 本博客所有文章除特别声明外,均采用 BY-NC-SA 许可协议。转载请注明出处!
暂无评论

发送评论 编辑评论


				
|´・ω・)ノ
ヾ(≧∇≦*)ゝ
(☆ω☆)
(╯‵□′)╯︵┴─┴
 ̄﹃ ̄
(/ω\)
∠( ᐛ 」∠)_
(๑•̀ㅁ•́ฅ)
→_→
୧(๑•̀⌄•́๑)૭
٩(ˊᗜˋ*)و
(ノ°ο°)ノ
(´இ皿இ`)
⌇●﹏●⌇
(ฅ´ω`ฅ)
(╯°A°)╯︵○○○
φ( ̄∇ ̄o)
ヾ(´・ ・`。)ノ"
( ง ᵒ̌皿ᵒ̌)ง⁼³₌₃
(ó﹏ò。)
Σ(っ °Д °;)っ
( ,,´・ω・)ノ"(´っω・`。)
╮(╯▽╰)╭
o(*////▽////*)q
>﹏<
( ๑´•ω•) "(ㆆᴗㆆ)
😂
😀
😅
😊
🙂
🙃
😌
😍
😘
😜
😝
😏
😒
🙄
😳
😡
😔
😫
😱
😭
💩
👻
🙌
🖕
👍
👫
👬
👭
🌚
🌝
🙈
💊
😶
🙏
🍦
🍉
😣
Source: github.com/k4yt3x/flowerhd
颜文字
Emoji
小恐龙
花!
上一篇
下一篇